Triangle abc has vertices at A(-6, 3) , B(3, 5) and C(1, -4). Show that triangle abc is isosceles and find the area of this triangle.

Answers

Answer 1

Using the formula for the distance between two points, it is found that:

Since sides AB and BC have the same length, which is different of side AC, the triangle is an isosceles triangle.The area is of 38.63 units squared.

What is the distance between two points?

Suppose that we have two points, [tex](x_1,y_1)[/tex] and [tex](x_2,y_2)[/tex]. The distance between them is given by:

[tex]D = \sqrt{(x_2-x_1)^2+(y_2-y_1)^2}[/tex]

In this problem, we use the formula to find the lengths for each side of the triangle.

For example, side AB has length given by the distance between (-6,3) and (3,5), hence:

[tex]AB = \sqrt{(-6 - 3)^2 + (3 - 5)^2} = 9.22[/tex]

Side AC has length given by:

[tex]AC = \sqrt{(-6 - 1)^2 + (3 - (-4))^2} = 9.9[/tex]

Side BC has length given by:

[tex]BC = \sqrt{(3 - 1)^2 + (5 - (-4))^2} = 9.22[/tex]

Since sides AB and BC have the same length, which is different of side AC, the triangle is an isosceles triangle.

What is the area of a triangle?

The area of a triangle is given by half the length of the base multiplied by the height.

The midpoint of side BC, of length of 9.22 units, is of (2,0.5). The height is the distance of this point from point A, hence:

[tex]d = \sqrt{(-6 - 2)^2 + (3 - 0.5)^2} = 8.38[/tex]

Hence the area is:

A = 0.5 x 9.22 x 8.38 = 38.63 units squared.

More can be learned about the distance between two points at https://brainly.com/question/18345417

#SPJ1


Related Questions

16. Describe the type of solution for the linear system of
equations given below.
2x + 3y = 15
6y=-4x + 12
F.
no solution
G. infinite solutions
H.
one solution
J. two solutions

Answers

Answer:

Step-by-step explanation:

2x+3y=15

multiply by 2

4x+6y=30   ...(1)

6y=-4x+12

4x+6y=12    ...(2)

(1) and (2) represent parallel lines.

Hence no solution.

1. Spring Time Manufacturers produces a single product and the
company is trying to determine the effectiveness of their
pricing decisions. As a consultant, you have been asked to
develop cost functions that will assist in arriving at the
optimal price that will enable the company to maximize
profits. During the year, you were provided with the following
demand and costs functions for the product:
P = 485-25Q, where P is the unit selling price and Q is quantity
of units in thousands.
TC = 5Q² +95Q + 200, where TC is total costs in thousands of
dollars.

Required:
(a) Find the output at which profit is maximized.
(b) Find the optimal price that maximizes profit.
(c) Determine the optimal sales revenue.
(d) Calculate the maximum profit.

Answers

I believe is b hope I was able to help you

order the decimales from leaste to gratest: 72.5, 73.943, 72.1, 73.77,

43.2 43.219 42.1 42.59


38.507 38.507 38.4 28.23 39.5


71.743 71.3 71.3 72.43 72.5

Answers

The decimals would be ordered as:

72.1, 72.5, 73.77, 73.943

42.1, 42.59, 43.2, 43.219

28.23, 38.4, 38.507, 38.507, 39.5

71.3, 71.3, 71.743, 72.43, 72.5

How to Order Decimals?

To order decimals from the least to the greatest, first state the lowest value, then progress to the highest taking account of the figures that come immediately after each decimal point.

The decimals will be ordered as shown below:

72.1, 72.5, 73.77, 73.943

42.1, 42.59, 43.2, 43.219

28.23, 38.4, 38.507, 38.507, 39.5

71.3, 71.3, 71.743, 72.43, 72.5

Learn more about ordering decimals on:

https://brainly.com/question/20603315

#SPJ1

The magnitude, M, of an earthquake is represented by the equation M=2/3logE/E0 where E is the amount of energy released by the earthquake in joules and E0=10^4.4 is the assigned minimal measure released by an earthquake. Which shows a valid step in the process of calculating the magnitude of an earthquake releasing 2.5 • 10^15 joules of energy?

2.5•10^15 = 2/3logE/10^4.4
10^4.4=2/3logE/2.5•10^15
M=2/3log(9.95•10^9)
M=2/3log(2.55•10^10)
M=2/3log(9.95•10^10)

Answers

The magnitude of an earthquake releasing 2.5 * 10¹⁵ Joules of energy is 7.33

What is an equation?

An equation is an expression that shows the relationship between two or more numbers and variables.

Given that:

M = (2/3) * log (E/E₀)

Where M is the magnitude, E is the amount of energy and E₀ = 10^4..4

For E = 2.5 * 10¹⁵:

M = (2/3) * log (2.5 * 10¹⁵/10^4.4)

M = 7.33

The magnitude of an earthquake releasing 2.5 * 10¹⁵ Joules of energy is 7.33

Find out more on equation at: https://brainly.com/question/2972832

#SPJ1

Will mark brainliest
Find an equation of the tangent line to the function
y = 3x2
at the point P(1, 3).
Solution
We will be able to find an equation of the tangent line ℓ as soon as we know its slope m. The difficulty is that we know only one point, P, on ℓ, whereas we need two points to compute the slope. But observe that we can compute an approximation to m by choosing a nearby point
Q(x, 3x2)
on the parabola (as in the figure below) and computing the slope mPQ of the secant line PQ. [A secant line, from the Latin word secans, meaning cutting, is a line that cuts (intersects) a curve more than once.]

Answers

The equation of the tangent line to the quadratic function y = 3 · x² at the point (x, y) = (1, 3) is y = 6 · x - 3.

How to determine the equation of a line tangent to a quadratic equation by algebraic methods

Herein we must determine a line tangent to the quadratic equation y = 3 · x² at the point P(x, y) = (1, 3) by algebraic means. The slope of the line can be found by using the secant line formula and simplify the resulting expression:

m = [3 · (x + Δx)² - 3 · x²] / [(x + Δx) - x]

m = 3 · [(x + Δx)² - x²] / Δx

m = 3 · (x² + 2 · x · Δx + Δx ² -  x²) / Δx

m = 3 · (2 · x + Δ x)

If Δx = 0, then the equation of the slope of the tangent line is:

m = 6 · x

If we know that x = 1, then the slope of the tangent line is:

m = 6 · 1

m = 6

Lastly, we find the intercept of the equation of the line: (x, y) = (1, 3), m = 6

b = y - m · x

b = 3 - 6 · 1

b = - 3

The equation of the tangent line to the quadratic function y = 3 · x² at the point (x, y) = (1, 3) is y = 6 · x - 3.

To learn more on tangent lines: https://brainly.com/question/23265136

#SPJ1

ION 10
answered
out of 1.00
Flag
n
What is the likelihood of Jada investing with Bank JNC if the following holds under the following conditions?
..
there is a 75% chance Jada will invest if the economic conditions remain stable;
there is a 25% chance investing if economic conditions suffer a decline;
there is a 55% chance of investing if the economic conditions improve.
the chance the economic conditions remaining stable (S), declining (D) and improving (1) are 0.20, 0.40
and 0.40, respectively.
Select one:
O.a. 0.135
O b. 0.103
OC. 0.400
O d. 0.470

Answers

Answer:

a

Step-by-step explanation:

The function g(x) = 10x2 – 100x + 213 written in vertex form is g(x) = 10(x – 5)2 – 37. Which statements are true about g(x)? Select three options. The axis of symmetry is the line x = –5. The vertex of the graph is (5, –37). The parabola has a minimum. The parabola opens up. The value of a, when the equation is written in vertex form, is negative.

Answers

Answer:

The vertex of the graph is (5, -37) [see attached image]The parabola has a minimum [the coefficient of x² is positive]The parabola opens up [the coefficient of x² is positive]

All the correct statements are,

The vertex of the graph is (5, -37).

The parabola has a minimum.

The parabola opens up.

What is an expression?

Mathematical expression is defined as the collection of the numbers variables and functions by using operations like addition, subtraction, multiplication, and division.

We have to given that;

The function g(x) = 10x² - 100x + 213 written in vertex form is,

⇒ g(x) = 10(x – 5)² – 37.

Since, General equation is,

y = a (x - h)² + k

Where, (h, k) is vertex of parabola.

Hence, We get;

The vertex of the graph is (5, -37)

Since, the coefficient of x² is positive

Hence, The parabola has a minimum

And, The parabola opens up.

Thus, All the correct statements are,

The vertex of the graph is (5, -37).

The parabola has a minimum.

The parabola opens up.

Learn more about the mathematical expression visit:

brainly.com/question/1859113

#SPJ5

PreCalc work, Need help writing piecewise functions with graphs. Giving brainliest

Answers

Answer:

f(x) = 2 for x < -2

f(x) = -2x + 11 for x > 3

the equation y=ax describes the graph of a line.if the value of a is negative,the line

Answers

If the value of a is negative, the line is reflected across any of the axis

How to describe the line?

The equation is given as:

y = ax

The new line is given as

y = -ax

The above implies that y = ax is transformed to y = -ax

The transformation can be any of:

reflection across the x-axisreflection across the y-axis

Hence, if the value of a is negative, the line is reflected across any of the axis

Read more about reflection at:

https://brainly.com/question/26748988

#SPJ1

About % of the area under the curve of the standard normal distribution is between z = − 0.9 z = - 0.9 and z = 0.9 z = 0.9 (or within 0.9 standard deviations of the mean).

Answers

Using the normal distribution, it is found that 63.18% of the area under the curve of the standard normal distribution is between z = − 0.9 z = - 0.9.

Normal Probability Distribution

The z-score of a measure X of a normally distributed variable with mean [tex]\mu[/tex] and standard deviation [tex]\sigma[/tex] is given by:

[tex]Z = \frac{X - \mu}{\sigma}[/tex]

The z-score measures how many standard deviations the measure is above or below the mean. Looking at the z-score table, the p-value associated with this z-score is found, which is the percentile of X.

The area within 0.9 standard deviations of the mean is the p-value of Z = 0.9(0.8159) subtracted by the p-value of Z = -0.9(0.1841), hence:

0.8159 - 0.1841 = 0.6318 = 63.18%.

More can be learned about the normal distribution at https://brainly.com/question/4079902

#SPJ1

The two-way table shows the number of students in a class who like mathematics and/or science. Like Mathematics Do Not Like Mathematics Total Like Science 18 ? 38 Do Not Like Science 16 6 32 Total 34 26 70

Answers

The missing number is 20.

What is the missing number?

Subtraction is the mathematical operation that is used to find the difference between two or more numbers.

In order to find the missing number, subtract the total number of people who like science and mathematics from the total number of people who like science

38 - 18 = 20

To learn more about subtraction, please check: https://brainly.com/question/854115

#SPJ1

Marina has a pattern to make bows that requires 1/4 yard of ribbon for each bow. Part A: Fill in the table to show how many bows she can make from a given length of ribbon.​

Answers

the table complete is:

x      y

1       4

2      8

3      12

4      16

Where x is the ribbon length in yards and y is the number of bows she can make.

How to complete the table?

We know that Marina needs 1/4 yards of ribbon for each bow.

Then, with one yard of ribbon, she can make 4 bows, then the relation between y, the number of bows she can make, and x, the yards of ribbon that she has, is:

y = 4*x

Now we want to complete the table:

x           y

1

2

3

4

To do so, we just need to evaluate the above function.

when x = 1.

y = 4*1 = 4

When x = 2:

y = 4*2 = 8

when x = 3

y = 4*3 = 12

when x = 4

y = 4*4 = 16

Then the table complete is:

x      y

1       4

2      8

3      12

4      16

If you want to learn more about evaluating:

https://brainly.com/question/1719822

#SPJ1

Any ideas for this graph

Answers

It’s a parabola;
(X+1)(X+3) = 0

X intercepts are at x=1 and x=3
Y intercept is at y=3

It’s neither odd one even function.

look at the picture

Answers

Answer:

C

Step-by-step explanation:

x²-9x<-8

x²-9x+(-9/2)²<-8+(-9/2)²

(x-9/2)²<-8+81/4

(x-9/2)²<(-32+81)/4

(x-9/2)²<49/4

|x-9/2|<7/2

-7/2<x-9/2<7/2

add 9/2

9/2-7/2<x-9/2+9/2<7/2+9/2

2/2<x<16/2

1<x<8

Which equation represents the vertex form of the equation y = x² + 2x - 6?
y = (x + 2)² -
y = (x + 1)2 - 6
y = (x + 2)²-7
y = (x + 1)²-7

Answers

Answer:

Step-by-step explanation:

Steps

1] On the right, put brackets around the 1st 2 terms.

y = (x^2 + 2x) - 6

2] Divide the second term's coefficient by 2 and square the result.

2/2 = 1

3] Square the result

1^2 = 1

4] Add that inside the brackets

y = (x^2 + 2x + 1) - 6

5] Subtract 1 outside the brackets. The original equation is still there.

y = (x^2 + 2x + 1) - 6 - 1

y = (x^2 + 2x + 1) - 7

6] What is inside the brackets is a perfect square.

y = (x + 1)^2 - 7

Answer D

please no scam please answer ASAP 30 points need steps​

Answers

QUE :

75 - [5 + 3 of (25 - 2 × 10)]

= 75 - [5 + 3 of ( 25 - 20)]

= 75 - [5 + 3 of 5]

= 75 - [5 + 15]

= 75 - 20

= 55

________________________________

HOPE IT HELPS

MARK BRAINLIEST!

FOLLOW ME!

!!HELP! 1-8 PLEASE ANSWERS ONLY PLEASE

Answers

Distributive Property :

[tex]\boxed {a(bx + c) = a(bx) + a(c)}[/tex] or

[tex]\boxed {(ax + b)(cx + d) = acx^{2} + bcx + adx + bd}[/tex]

Question 1 :

6(4v + 1)6(4v) + 6(1)24v + 6

Question 2 :

5(8r² - r + 6)5(8r²) - 5(r) + 5(6)40r² - 5r + 30

Question 3 :

(2x - 2)(7x - 4)2x(7x) - 2(7x) - 2x(4) - 2(-4)14x² - 14x - 8x + 814x² - 22x + 8

Question 4 :

(6a - 7)(3a - 8)(6a)(3a) - 7(3a) + (6a)(-8) - 7(-8)18a² - 21a - 48a + 5618a² - 69a + 56

Heights of men on a baseball team have a bell-shaped distribution with a mean of 176cm and a standard deviation of 5cm .Using the empirical rule,what is the approximate percentage of men between the following values?


% of the men are between 165cm and 186cm

Answers

95% men are between 165 cm and 186 cm.

What is the empirical rule?

The empirical rule is also referred to as the Three Sigma Rule or the 68-95-99.7 Rule.

z-score = (raw-score minus mean) / standard deviation.

z1 = (165-176)/5 = -2.2

z2 = (186-176)/5 = 2

The empirical rule tells us that about 95% of all values are within standard deviations of the mean,

so, 95% men are between 165 cm and 186 cm.

to know more please refer: https://brainly.com/question/10093236

#SPJ9

95% of men are between 165 cm and 186 cm.

What is the approximate percentage of men between the following values?

Given:

The heights of men on a baseball team have a bell-shaped distribution a mean of 176cm and a standard deviation of 5cm.

Find:

What is the approximate percentage of men between the following values?

Solution:

The empirical rule is also referred to as the three sigma rule or the 68-95-99.7

Rule:

z - score = (raw - score minus mean) / standard deviation.

z1 = (165-176)/5 = -2.2

z2 = (186-176)/5 = 2

The empirical rule tells us that about 95% of all values are within standard deviations of the mean.

So, 95% of men are between 165 cm and 186 cm.

To learn more about the empirical rules, refer to:

brainly.com/question/10093236

#SPJ9

A total of $5000 is invested: part at 7% and the remainder at 12%. How much is invested at each rate if the annual interest is $400?

Answers

The amount invested in the account that yields 7% interest is $4000.

The amount invested in the account that yields 12% interest is $1000.

What are the linear equations that represent the question?

a + b = 5000 equation 1

0.07a + 0.12b = 400 equation 2

Where:

a = amount invested in the account that yields 7% interest.

b =  amount invested in the account that yields 12% interest.

How much is invested at each rate?

Multiply equation 1 by 0.07

0.07a + 0.07b = 350 equation 3

Subtract equation 3 from equation 2

0.05b = 50

b = 50 / 0.05

b = 1000

Subtract 1000 from 5000: 5000 - 1000 = 4000

To learn more about linear functions, please check: https://brainly.com/question/26434260

#SPJ1

Please Please Please help with this math problem

Answers

The revenue as a function of x is equal to -x²/20 + 920x.The profit as a function of x is equal to -x²/20 + 840x - 6000.The value of x which maximizes profit is 8,400 and the maximum profit is $3,522,000.The price to be charged to maximize profit is $500.

How to express the revenue as a function of x?

Based on the information provided, the cost function, C(x) is given by 80x + 6000 while the demand function, P(x) is given by -1/20(x) + 920.

Mathematically, the revenue can be calculated by using the following expression:

R(x) = x × P(x)

Revenue, R(x) = x(-1/20(x) + 920)

Revenue, R(x) = x(-x/20 + 920)

Revenue, R(x) = -x²/20 + 920x.

Expressing the profit as a function of x, we have:

Profit = Revenue - Cost

P(x) = R(x) - C(x)

P(x) = -x²/20 + 920x - (80x + 6000)

P(x) = -x²/20 + 840x - 6000.

For the value of x which maximizes profit, we would differentiate the profit function with respect to x:

P(x) = -x²/20 + 840x - 6000

P'(x) = -x/10 + 840

x/10 = 840

x = 840 × 10

x = 8,400.

For the maximum profit, we have:

P(x) = -x²/20 + 840x - 6000

P(8400) = -(8400)²/20 + 840(8400) - 6000

P(8400) = -3,528,000 + 7,056,000 - 6000

P(8400) = $3,522,000.

Lastly, we would calculate the price to be charged in order to maximize profit is given by:

P(x) = -1/20(x) + 920

P(x) = -1/20(8400) + 920

P(x) = -420 + 920

P(x) = $500.

Read more on maximized profit here: https://brainly.com/question/13800671

#SPJ1

Solve. −4 3/4=x−1 1/5 What is the solution to the equation? Enter your answer as a simplified mixed number in the box.

plssssss helppp

Answers

Answer:  x= -3 11/20 Decimal form: x=-3.55

Step-by-step explanation:

help this is affecting my grade i need help pls i beg of you

Answers

it is -4.5 or four and a half
the answer is -5 1/2

Evaluate function expressions

Answers

Answer:

Your answer is -24.

Step-by-step explanation:

Given information.

The graph of f(x) and g(x)

Solving for

-6 * f(3) - 6 * g(-1) = ?

think of f(x) = y as x is the input and y is the output.

Input a value of x into f(x) or g(x) gets us a y value.

Looking at the graph of f(3) = -2 and the graph of g(-1) = 6

Now substitute that and solve.

-6 * -2 -6 * 6 = 12 - 36 = -24

Answer: -24

Step-by-step explanation:

We should first find the outputs to the functions f and g with inputs 3 and -1 respectively. We can do this by looking at the graph and finding the y value for each desired x value.

Thus, we can see that f(3) = -2 and g(-1) is 6.

We can replace these into the expression to get

[tex]-6*-2-6*6[/tex]

We should first multiply, so we get

[tex]12 -36[/tex]

[tex]-24[/tex]

Hence, the answer is -24.

PLS ANSWER THESE QUESTIONS
I WILL MARK BRAINLIEST

Answers

The linear function y = 3 · w - 1 represents the number of sea shells found in each week.

The speed of the driven gear is 180 rounds per minute.

How to use direct and inverse relationships to analyze situations

In the first problem we have an example of linear progression, in which the number of sea shells is increased linearly every week. After a quick analysis, we conclude that the linear function y = 3 · w - 1, a kind of direct relationship.

In the second problem, we must an inverse relationship to determine the speed of the driven gear. Please notice that the speed of the gear is inversely proportional to the number of teeths. Then, we proceed to calculate the speed:

[tex]\frac{v_{1}}{v_{2}} = \frac{N_{2}}{N_{1}}[/tex]

If we know that [tex]v_{2} = 60\,rpm[/tex], [tex]N_{2} = 60[/tex] and [tex]N_{1} = 20[/tex], then the speed of the driven gear is:

[tex]v_{1} = v_{2}\times \frac{N_{2}}{N_{1}}[/tex]

[tex]v_{1} = 60\,rpm \times \frac{60}{20}[/tex]

[tex]v_{1} = 180\,rpm[/tex]

The speed of the driven gear is 180 rounds per minute.

To learn more on inverse relationships: https://brainly.com/question/4147411

#SPJ1

The graph of the discrete probability to the right represents
the number of live births by a mother 40 to 44 years old
who had a live birth in 2015. Complete parts (a) through (d)
below.

0.30-
0.25-
0.20
0.15
0.10
0.05
0.00
0
0.235
1
0.270
2
0784
113 0101
-4426-0004 0.045
3
6
Number of Live Births
(a) What is the probability that a randomly selected 40- to 44-year-old mother who had a live birth in 2015 has had her fourth live birth in that year?
(Type an integer or a decimal)
(b) What is the probability that a randomly selected 40- to 44-year-old mother who had a live birth in 2015 has had her fourth or fifth live birth in that year?
(Type an integer or a decimal.)
(c) What is the probability that a randomly selected 40- to 44-year-old mother who had a live birth in 2015 has had her sixth or more live birth in that year?
(Type an integer or a decimal)
(d) If a 40-to 44-year-old mother who had a live birth in 2015 is randomly selected, how many live births would you expect the mother to have had?

Answers

The values of the probabilities are

The probabilities are 0.109, 0.202, 0.106The expected number of births is 3

How to determine the probabilities?

The image that completes the question is added as an attachment

The probability of having her fourth live birth in that year?

From the attached graph, we have:

P(x) = 0.109 when x = 4

Hence, the probability is 0.109

The probability of having a live birth in her fourth or fifth live birth in that year?

From the attached graph, we have:

P(x) = 0.109 when x = 4

P(x) = 0.093 when x = 5

So, we have:

P(4 or 5) = 0.109 + 0.093

Evaluate

P(4 or 5) = 0.202

Hence, the probability is 0.202

The probability of having a live birth in her sixth or more live birth in that year?

This is represented as:

P(x >= 6)

From the attached graph, we have:

P(x) = 0.022 when x = 6

P(x) = 0.036 when x = 7

P(x) = 0.048 when x = 8

So, we have:

P(x >= 6) = 0.022 + 0.036 + 0.048

Evaluate

P(x >= 6) = 0.106

Hence, the probability is 0.106

How many live births would you expect the mother to have had?

This is calculated as:

[tex]E(x) = \sum x * P(x)[/tex]

So, we have:

E(x) = 0.234 * 1 + 0.291 * 2 + 0.167 * 3 + 0.109 * 4 + 0.093 * 5 + 0.022 * 6 + 0.036 * 7 + 0.048 * 8

Evaluate

E(x) = 2.986

Approximate

E(x) = 3

Hence, the expected number of births is 3

Read more about probability at:

https://brainly.com/question/25870256

#SPJ1

lim
x →1+. 1- x/x² - 1

Answers

Answer:  [tex]\displaystyle \boldsymbol{-\frac{1}{2}}[/tex]

================================================

Work Shown:

[tex]\displaystyle L = \lim_{\text{x}\to 1^{+}} \frac{1-\text{x}}{\text{x}^2-1}\\\\\\\displaystyle L = \lim_{\text{x}\to 1^{+}} \frac{-(\text{x}-1)}{(\text{x}-1)(\text{x}+1)}\\\\\\\displaystyle L = \lim_{\text{x}\to 1^{+}} \frac{-1}{\text{x}+1}\\\\\\\displaystyle L = \frac{-1}{1+1}\\\\\\\displaystyle L = -\frac{1}{2}\\\\\\[/tex]

In the second step, I used the difference of squares rule to factor.

The (x-1) terms cancel which allows us to plug in x = 1. We plug this value in because x is approaching 1 from the right side.

Find and sketch the domain of f(X,y) = 1/√x^2-y

Answers

Answer:

Step-by-step explanation:

The definition of a Domain in math is all the possible input values that go into the function, so we will have to find all the valid values that can go into the function

The function given is [tex]F(x, y)=\frac{1}{\sqrt{x^2-y} }[/tex] , the denominator cannot be 0.
So we set up the equation
[tex]\sqrt{x^2-y} \neq 0[/tex]
[tex]\sqrt{x^2-y}^{2} \neq 0x^2-y\neq 0x^2\neq y[/tex]

And that the the square root needs to be more than 0.
[tex]\sqrt{x^2-y}\geq 0[/tex]
[tex]x^2-y\geq 0[/tex]
[tex]x^2\geq y[/tex]

So we can conclude that all values of [tex]x^{2}[/tex] must be greater y

That means that our domain is all X values greater than[tex]\sqrt{y}[/tex]

Select the correct answer from each drop-down menu. The options are: The ratio of the heights is 1 : 2.5 1 : 5 1 : 10 1 : 25 The ratio of the surface areas is 1 : 5 1 : 10 1 : 25 1 : 125 The ratio of the volumes is 1 : 5 1 : 10 1 : 25 1 : 125.

Answers

Using proportions, it is found that:

The ratio of heights is of 1:5.The ratio of surface areas is of 1:25.The ratio of volumes is of 1:125.

What is a proportion?

A proportion is a fraction of a total amount, and the measures are related using a rule of three.

The heights are measured in units, hence the ratio is:

[tex]r = \frac{5}{25} = \frac{1}{5}[/tex]

The surface areas are measured in units squared, hence the ratio is:

(1:5)² = 1:25.

The volumes are measured in cubic units, hence the ratio is:

(1:5)³ = 1:125.

More can be learned about proportions at https://brainly.com/question/24372153

#SPJ1

Given: F(x) = 3xˆ2+ 1, G(x) = 2x-3, H(x) = x F(-2) =

heeeelp

Answers

Answer:

firstly,need to know domain and range

Simplify. x^2+5x-/14 x²+8x+7​

Answers

please send a picture of it

the equation seems a lil bit complicated

Other Questions
Mariana gets $15 per hour working at McDonald's and Jayvon gets $15 per hour as well. Jayvon had $50 in thebank to start. After how many weeks will Jayvon and Mariana have the same amount of money in their bankaccounts? Why would China be attractive to people who would want to conquer them? PLSSS HELPPPP I WILLL GIVE YOU BRAINLIEST!!!!!! PLSSS HELPPPP I WILLL GIVE YOU BRAINLIEST!!!!!! PLSSS HELPPPP I WILLL GIVE YOU BRAINLIEST!!!!!! PLSSS HELPPPP I WILLL GIVE YOU BRAINLIEST!!!!!!PLSSS HELPPPP I WILLL GIVE YOU BRAINLIEST!!!!!! PLSSS HELPPPP I WILLL GIVE YOU BRAINLIEST!!!!!! PLSSS HELPPPP I WILLL GIVE YOU BRAINLIEST!!!!!!The Explorers' Legacy - After a wet and hungry winter in Oregon, the explorers headed home and finally returned to St. Louis in September 1806, two years and four months after setting out. Lewis proudly wrote to Jefferson, In obedience to our orders, we have penetrated the Continent of North America to the Pacific Ocean.Lewis and Clark had good reason to be proud. Although they had not found the Northwest Passagefor it did not existthey had traveled some 8,000 miles and had mapped a route to the Pacific. They had established good relations with western Indians. Most of all, they had brought back priceless information about the West and its peoples.Would you consider the expedition a success? Why or why not? In the central processing unit (CPU).A) Causes data to be input into the systemB) Responsible for the logical order of processing and converting data.C) BothD) None of the above it says priya's family exchanged 250 dollars for 4,250 pesos. Riya bought a sweater for 510 pesos. how many did the sweater cost? In which part of the world did the Enlightenment begin? A.EuropeB.AsiaC.The United States D.South America Did immigrant children get to go outside and play? Is nitric acid a strong or weak acid. Explain your answer. Which branch of the government raises revenue through taxes and other levies?JudicialNoneLegislativeExecutive Can someone help me with this problem! I WILL GIVE A BRAINIEST!!! I need it asap PLEASE. A class of religious officials that includes priests, pastors, bishops, and monks.A. PapacyB. ClergyC. ConcordatPLEASE HELP MEH 3 x + 4 = 3 x + 4 3 x + 4 = 3 x + 4 Which excerpt from We ve Got a Job expresses the author's viewpoint about segregation in Birmingham?Abe"My parents were involved from the point that I could remember My church was involved You were there andjust a part of it.""She would no longer just attend meetings. Somehow, she would act on her own.""Separate accommodations for the white and colored races were allowable, as long as the accommodations foreach group were equal.""There were several things wrong-above all, the concept of 'separate but equal itself. Separation on the basis ofrace is simply unjust." Vitamin D is important for the metabolism of calcium and exposure to sunshine is an important source of vitamin D. A researcher wanted to determine whether osteoperosis was associated with a lack of exposure to sunshine. He selected a sample of 250 women with osteoperosis and an equal number of women without osteoperosis. The two groups were matched - in other words they were similar in terms of age, diet, occupation, and exercise levels. Histories on exposure to sunshine over the previous twenty years were obtained for all women. The total number of hours that each woman had been exposed to sunshine in the previous twenty years was estimated. The amount of exposure to sunshine was compared for the two groups.Required:a. Determine whether this is a controlled experiment or an observational study. b. Identify the treatment variable. Bluebeard: what test did he give his wife and whats causes his wife to fail the test? What are the three types of plate boundaries? What is the direction of movement at each boundary? Theresa has $708 in her account and wishes to throw a party. She plans to include the following:PurchaseCost ($)Decorations76.92Catering150.66Musicians242.15Refreshments123.36Venue260.47When Theresa makes the calculations, she finds that she cannot afford everything that she would like to have. What is the cheapest aspect of the party that she can eliminate to keep from overdrawing her account? Phenotype of offspring:-?Genotypes of offspring:-?Lol I need help In this figure, angles R and S are complementary. Find the measure of angle S.S62RPLEASE HELP ASAP 2. This month I have to write a thesis, do a book report and a poem. (Compound) 3. It is a post-graduate degree achieving which will neither be easy nor cheap. (Simple) 4. They need to buy potatoes, apples, and cherries from the market. (Compound) tell the answers plzzzzzzzzzzzzzzzzzzzzzzzzzzzzzzzzzzzzzzzzzzzzzzzzzz